3

I am struggling to obtain the value of the following definite integral: $$g_1(\underline{\xi}) = \int_{\mathcal{B}_+(R)} e^{i \underline{\xi} \cdot \underline{X} } \mathrm{d}\underline{X} $$ with $\underline{\xi}$ a vector of $\mathbb{R}^3$ and $\mathcal{B}_+(R) = \{ \underline{X} \in \mathbb{R}^3 : \|\underline{X}\|_2 \leq R, X_3 \geq 0\}$ a demi-ball. Note that there is no guarantee that such a result exists.

I already computed the following integral through spherical coordinates associated with a Cartesian frame of reference in which $\underline{\xi} = \{ 0, 0, \xi\}$: $$g_0(\underline{\xi}) = \int_{\mathcal{B}(R)} e^{i \underline{\xi} \cdot \underline{X} } \mathrm{d}\underline{X} = \frac{4\pi}{\xi^3}\left(\sin(R\xi)-R\xi \cos(R\xi)\right) $$
Unfortunately, I get nowhere with such an method on $g_1$. So far I have only noticed that $g_1(\xi) + \overline{g_1(\xi)} = g_1(\xi) + g_1(-\xi) = g_0(\xi)$ so the only unknown lie in the imaginary part of $g_1$.

CNS
  • 69
  • 1
    Please take a look at https://math.stackexchange.com/questions/489391/fourier-transform-of-the-indicator-of-the-unit-ball – Stefan Lafon Jan 07 '22 at 15:52
  • 1
    @StefanLafon The solution of the question you point at rely on the same rotation trick as the computation of $g_0$ I mentionned (in fact it is the computation of $g_0$ for any dimension $n$), trick that is ineffective for $g_1$. Did I miss something? – CNS Jan 07 '22 at 16:11
  • 1
    Oh sorry, I was confused. – Stefan Lafon Jan 07 '22 at 18:05
  • crossposted at https://mathoverflow.net/q/416545/11260 – Carlo Beenakker Feb 19 '22 at 23:03

1 Answers1

1

It would be nice if this was listed in some table of Fourier transforms. I looked at Messiah's book in QM and Pauli's book on Chemistry. Neither of those had anything. Mathematica does not simplify the integral. I did not see much in Stein and Weiss. You could try to calculate the integral of all the spherical harmonics over that set, and try to obtain a series from that. In principle, I believe that you can get it down to a 1d integral to put into Mathematica, by direct calculus techniques. But I think Mathematica would not do that integral. Before I state those calculations, it feels to me like a physics approach would be to try to do some Gaussian integrals.

Define $F(\kappa,\lambda) = |\mathbb{S}^+|^{-1} \int_{\mathbb{S}^+} e^{i (\kappa x + \lambda z)}\, d^2S(x,y,z)$ for $\mathbb{S}^+$ being the restriction of the 2d sphere $\mathbb{S}$ to the upper half-space $\{(x,y,z) : z>0\}$ and $d^2S(x,y,z)$ being the surface measure on the sphere. So $|\mathbb{S}^+|=2\pi$. Then something that would be easy to calculate is the average $\int_0^{\infty} \sqrt{\frac{2}{\pi}}\, F(r\kappa,r\lambda) r^2 e^{-r^2/2}\, dr$ because that will give Gaussian integrals $$ \frac{2}{(2\pi)^{3/2}}\, \int_{\mathbb{R}^3} \left(e^{i \kappa x} e^{-x^2/2}\right) e^{-y^2/2} \left(e^{i \lambda z} e^{-z^2/2} \mathbf{1}_{(0,\infty)}(z)\right)\, dx\, dy\, dz $$ which equals $e^{-(\kappa^2+\lambda^2)/2} \left(1 + \operatorname{erf}\left(\frac{i \lambda}{\sqrt{2}}\right)\right)$ according to Mathematica. Now by symmetry and scaling, you have $\int_0^{\infty} \frac{\sqrt{2}}{\sqrt{\pi}\, \sigma^3} F(r\kappa,r\lambda) r^2 e^{-r^2/(2\sigma^2)}\, dr$ equals $e^{-\sigma^2(\kappa^2+\lambda^2)/2} \left(1 + \operatorname{erf}\left(\frac{i \lambda \sigma}{\sqrt{2}}\right)\right)$ for each $\sigma>0$. But I do not see how to go back from there to get the Dirac-delta function in $r$, which is what I feel a physicist would try to do. (It might be basic and I am forgetting.)

You can analytically continue the Fourier transform to elements $\underline{\xi} \in \mathbb{C}^3$. But generally for an analytic function on $\mathbb{C}$ there is no unique way to go from the real part of the function on $\mathbb{R}$ to recover the whole function.

In terms of getting down to 1 integral, part of it uses the incomplete Gamma function, which is just another way of writing a particular integral, and part of it uses the Bessel function (which is another way of writing a different integral). But in case it is useful, here is part of it:Set $R=1$. You can rescale the radius of the ball into the modulus of $\underline{\xi}$. In terms of the Bessel function, $\pi^{-1} \int_0^{\pi} e^{i t \cos(\theta)}\, d\theta = J_0(t)$. Therefore, using cylindrical coordinates $\xi_{\rho} = \sqrt{\xi_1^2+\xi_2^2}$ for $\underline{\xi}=(\xi_1,\xi_2,\xi_3)$, your integral is $$ \int_0^1 \left( \int_0^{\pi/2} J_0\big(\xi_\rho r \sin(\theta)\big) \exp\left(i \xi_3 r \cos(\theta)\right) \sin(\theta)\, d\theta\right)\, dr $$ But Mathematica does not give me an integral for this. If we define $w=r \sin(\theta)$ and use the coordinates $r$ and $w$, so that $\sin(\theta)=w/r$, $dw=r \cos(\theta)\, d\theta$ (holding $r$ fixed) and $r \cos(\theta) = \sqrt{r^2-w^2}$ then we get $$ \int_0^1 \left( \int_0^{r} J_0\big(\xi_\rho w\big) \exp\left(i \xi_3 \sqrt{r^2-w^2}\right) \frac{w/r}{\sqrt{r^2-w^2}}\, dw\right)\, dr $$ Then we could interchange the order of the integral $$ \int_0^1 J_0\big(\xi_\rho w\big) \left( \int_w^{1} \exp\left(i \xi_3 \sqrt{r^2-w^2}\right) \frac{w/r}{\sqrt{r^2-w^2}}\, dr\right)\, dw $$ Now Mathematica will evaluate $\int_0^1 e^{i a x} (b^2+x^2)^{-1}\, dx$ for $a,b \in \mathbb{R}$ in terms of elementary functions and $\Gamma(0,z)=\int_z^{\infty} t^{-1} e^{-t}\, dt$. The expression is somewhat complicated. But using $t = \sqrt{r^2-w^2}/\sqrt{1-w^2}$, you can get $$ \int_w^{1} \exp\left(i \xi_3 \sqrt{r^2-w^2}\right) \frac{w/r}{\sqrt{r^2-w^2}}\, dr\, =\, \frac{w}{\sqrt{1-w^2}}\, \int_0^1 \frac{e^{i a t}}{b^2+t^2}\, dt\, , $$ where $a = \xi_3 \sqrt{1-w^2}$ and $b = w/\sqrt{1-w^2}$. Then you would multiply the result of whatever comes out of Mathematica by $J_0(\xi_\rho w)$ and you would have a single integral over $w$.

  • By the way, the comment about what physicists would do is in cases that are somewhat similar, but where one considers large dimensions, possibly along with some asymptotics in $R$. Then the Gaussian approximation to a Dirac-delta function or a step function may be sufficient to determine leading-order asymptotics. (For example, that is the type of thing done, although in large dimensions and with an asymptotics in R for the spherical model of a spin glass by Kosterlitz, Thouless and Jones.) – Shannon Starr Jan 07 '22 at 21:24
  • 1
    Thanks for your thoughts. Speaking of Dirac distribution, $$\underset{a\rightarrow 0}{lim} \frac{1}{\sqrt{\pi}a}\int_0^{\infty} F(r\kappa,r\lambda) r^2 e^{-r^2/2 -((r-R)/a)^2}, dr = F(R\kappa,R\lambda)R^2 e^{-R^2/2}$$ but the $e^{-2Rr/a^2} = e^{-2R\sqrt{x^2 + y^2 + z^2}/a^2} $ factor unfortunately thwarts the gaussian integral in Cartesian frame. Apart from functions that converge towards it, I'm not sure how you can include Dirac distribution. Despite having read the paper you mention in your previous comment, I fail to grasp the inspiration that can be drawn from its resolution. – CNS Jan 10 '22 at 12:38
  • 1
    You're right about the 1-dimension integral; Mathematica won't calculate it. For others thta may read us: – CNS Jan 10 '22 at 13:50
  • 1
    For others that may read us: $$\int_0^1 \frac{e^{i a t}}{b^2+t^2}, dt = \frac{i}{2b}[e^{-ab}(-\Gamma(0,-ab)+\Gamma(0,-a(b + i))-\ln(1-ib)+\ln(-ib)-\ln(-ab)+\ln(-a(b+i)) + e^{ab}(\Gamma(0,ab)+\Gamma(0,a(b -i))-\ln(1+ib)+\ln(ib)-\ln(ab)+\ln(a(b-i))]$$ – CNS Jan 10 '22 at 13:56
  • 1
    @CNS you are right. There is no inspiration to draw from the Gaussian calculation. It just shows up in various different applications. If you want asymptotics in $R$ or if you have different dimensions going to $\infty$ or both, then you get some utility. I will keep looking for a table with that information. It seems like it should be written somewhere. Your calculation in terms of the incomplete Gamma is absolutely right, too. Thanks for writing that down! – Shannon Starr Jan 10 '22 at 14:54
  • As for plane-wave expansion (which is close from what you suggested with spherical harmonics but maybe simpler because there is only one sum): $$e^{i \underline{\xi} \cdot \underline{X} } = \sum_{m=0}^{\infty} (2m+1)i^m j_m(\xi r)P_m(\underline{\xi} \cdot \underline{X})$$ with $P_m$ the m-th Legendre polynomial and $j_m$ the m-th spherical Bessel function. However $\int_{\mathbb{S}^+} P_m(\underline{\xi} \cdot \underline{X})\mathrm{d}X$ seem really painful to compute (while it is $0$ for $m>0$ on $\mathbb{S}$ !). – CNS Jan 10 '22 at 17:15